You wish to purchase a new car and can afford monthly repayments of up to $250 per month. You see a car you like for $12,000 and are offered financing to secure the purchase. The loan lasts for 5 years and the annual interest rate is 17.9%. Can you afford to purchase the car on finance?

Answers

Answer 1

Answer:

Since the finance’s monthly required payment of $304.07 is greater than the $250 monthly that you can afford, this implies that you CANNOT afford to purchase the car on finance.

Step-by-step explanation:

The monthly required payment of the finance can be calculated using the formula for calculating the present value of an ordinary annuity as follows:

PV = M * ((1 - (1 / (1 + r))^n) / r) …………………………………. (1)

Where;

PV = Present value or finance amount = $12,000

M = Finance’s monthly required payment = ?

r = Monthly interest rate = Annual interest rate / 12 = 17.9% / 12 = 0.179 / 12 = 0.0149166666666667

n = number of months = Number of years * 12 = 5 * 12 = 60

Substitute the values into equation (1) and solve for M, we have:

$12,000 = M * ((1 - (1 / (1 + 0.0149166666666667))^60) / 0.0149166666666667)

$12,000 = M * 39.464764662266

M = $12,000 / 39.464764662266 = $304.07

Since the finance’s monthly required payment of $304.07 is greater than the $250 monthly that you can afford, this implies that you CANNOT afford to purchase the car on finance.


Related Questions

prime numbers that can be expressed as a sum and difference of 2 prime numbers
ASAP

Answers

Given:

Prime numbers can be expressed as a sum and difference of 2 prime numbers.

To find:

The prime numbers that can be expressed as a sum and difference of 2 prime numbers.

Solution:

Prime numbers are the positive integers which are greater than 1, divisible by only 1 and itself.

Prime numbers are 2, 3, 5, 7, 11, 13, 17, 19,... .

Only 2 is the even prime number.

The sum and difference of two odd integers is always an even number. So, we need to take one even prime number and one odd prime number to add and subtract the numbers to get a prime number.

[tex]2+3=5[/tex]

[tex]7-2=5[/tex]

Therefore, 5 is the only prime number that can be expressed as a sum and difference of 2 prime numbers.

A number cube has faces numbered 1 to 6.

What is true about rolling the number cube one time?

Answers

Answer:

All numbers have an equal chance of being landed on.

can someone please help me solve this? thank you!:)

Answers

First, we need to set up our two equations. For the picture of this scenario, there is one length (L) and two widths (W) because the beach removes one of the lengths. We will have a perimeter equation and an area equation.

P = L + 2W

A = L * W

Now that we have our equations, we need to plug in what we know, which is the 40m of rope.

40 = L + 2W

A = L * W

Then, we need to solve for one of the variables in the perimeter equation. I will solve for L.

L = 40 - 2W

Now, we can substitute the value for L into L in the area equation and get a quadratic equation.

A = W(40 - 2W)

A = -2W^2 - 40W

The maximum area will occur where the derivative equals 0, or at the absolute value of the x-value of the vertex of the parabola.

V = -b/2a

V = 40/2(2) = 40/4 = 10

Derivative:

-4w - 40 = 0

-4w = 40

w = |-10| = 10

To find the other dimension, use the perimeter equation.

40 = L + 2(10)

40 = L + 20

L = 20m

Therefore, the dimensions of the area are 10m by 20m.

Hope this helps!

Answer:

Width: 10 m

Length: 20 m

Step-by-step explanation:

Hi there!

Let w be equal to the width of the enclosure.

Let l be equal to the length of the enclosure.

1) Construct equations

[tex]A=lw[/tex] ⇒ A represents the area of the enclosure.

[tex]40=2w+l[/tex] ⇒ This represents the perimeter of the enclosure. Normally, P=2w+2l, but because one side isn't going to use any rope (sandy beach), we remove one side from this equation.

2) Isolate one of the variables in the second equation

[tex]40=2w+l[/tex]

Let's isolate l. Subtract 2w from both sides.

[tex]40-2w=2w+l-2w\\40-2w=l[/tex]

3) Plug the second equation into the first

[tex]A=lw\\A=(40-2w)w\\A=40w-2w^2\\A=-2w^2+40w[/tex]

Great! Now that we have a quadratic equation, we can do the following:

Solve for its zeros/w-intercepts.Take the average of the zeros to find the w-variable of the vertex. (The area (A) in relation to the width of the swimming area (w) is what we've established in this equation, and the area (A) is greatest at the vertex. Finding the value of w of the vertex will tell us what the width needs to be for the area to be at a maximum.)Plug this w value into one of the equations to solve for l

4) Solve for w

[tex]A=-2w^2+40w[/tex]

Factor out -2w

[tex]A=-2w(w-20)[/tex]

For A to equal 0, w=0 or w=20.

The average of 0 and 20 is 10, so the width that will max the area is 10 m.

5) Solve for l

[tex]40=2w+l[/tex]

Plug in 10 as w

[tex]40=2(10)+l\\40=20+l\\l=20[/tex]

Therefore, the length of 20 m will max the area.

I hope this helps!

the cost of a popular toy is $19.99 , Due to a limited supply , the store manager marks the price of the toy up by 15%. what is the new price ? HURRY PLEASE .

Answers

$22.99

Step-by-step explanation:

19.99 increased by 15% ≈ 22.99

Absolute change (actual difference):

22.99 - 19.99 ≈ 3

The difference is $3

What is the value of x?


[tex] \frac{4}{5} x - \frac{1}{10} = \frac{3}{10} [/tex]

Answers

Answer:

x = [tex]\frac{1}{2}[/tex]

Step-by-step explanation:

Given

[tex]\frac{4}{5}[/tex] x - [tex]\frac{1}{10}[/tex] = [tex]\frac{3}{10}[/tex]

Multiply through by 10 ( the LCM of 5 and 10 ) to clear the fractions

8x - 1 = 3 ( add 1 to both sides )

8x = 4 ( divide both sides by 8 )

x = [tex]\frac{4}{8}[/tex] = [tex]\frac{1}{2}[/tex]

What is the value of x?
1/2(x+6)=18

Answers

Answer:

30

Step-by-step explanation:

Multiply by 2 to 18, x+6=36

subtract 6 x=30

x=30

An arithmetic sequence follows a specific pattern where a number is _____________ to get from one term to the next.

Answers

Answer:

Added

Step-by-step explanation:

For example, take the arithmetic sequence of 3, 5, 7, 9...

Here, the number that gets us from the first number to the second (from 3 to 5), is 2. Meaning, we need to add 2 to 3 in order to get 5. I believe that makes "addition" or "added" the answer. Please let me know if I misunderstood and I will change my answer! Thank you!

Answer:

added or subtracted

Step-by-step explanation:

someone help me for this algebra task please

Answers

Answer:

200

Step-by-step explanation:

Substitute 15 for y

[tex] \frac{1}{5} x - \frac{2}{3} (15) = 30[/tex]

[tex] \frac{1}{5} x - 10 = 30[/tex]

[tex] \frac{1}{5} x = 40[/tex]

[tex]x = 200[/tex]

What is the answer to 3/4 x 1/2

Answers

What is the answer to 3/4 x 1/2?

Answer : 0.375

Answer:

3/8

Step-by-step explanation:

What is the solution of 5/2x-7=3/4x+14

Answers

Answer:

D. x = 12

Step-by-step explanation:

if you plug 12 in

the solutoon says 23 = 23 which is correct

Answer:D 12

Step-by-step explanation:

I took the test

Which inequality matches the graph?

X, Y graph. X range is negative 10 to 10, and y range is negative 10 to 10. Dotted line on graph has positive slope and runs through negative 3, negative 8 and 1, negative 2 and 9, 10. Above line is shaded.

−2x + 3y > 7
2x + 3y < 7
−3x + 2y > 7
3x − 2y < 7

Answers

Given:

The dotted boundary line passes through the points (-3,-8), (1,-2) and (9,10).

Above line is shaded.

To find:

The inequality for the given graph.

Solution:

Consider any two points on the line. Let the two points are (1,-2) and (9,10). So, the equation of the line is:

[tex]y-y_1=\dfrac{y_2-y_1}{x_2-x_1}(x-x_1)[/tex]

[tex]y-(-2)=\dfrac{10-(-2)}{9-1}(x-1)[/tex]

[tex]y+2=\dfrac{10+2}{8}(x-1)[/tex]

[tex]y+2=\dfrac{12}{8}(x-1)[/tex]

[tex]y+2=\dfrac{3}{2}(x-1)[/tex]

Multiply both sides by 2.

[tex]2(y+2)=3(x-1)[/tex]

[tex]2y+4=3x-3[/tex]

[tex]2y-3x=-3-4[/tex]

[tex]-3x+2y=-7[/tex]

Above line is shaded and the boundary line is a dotted line. So, the sign of inequality must be >.

[tex]-3x+2y>-7[/tex]

This inequality is not in the equations. So, multiply both sides by -1 and change the inequality sign.

[tex](-3x+2y)(-1)<-7(-1)[/tex]

[tex]3x-2y<7[/tex]

Therefore, the correct option is D.

The value of -9 is __ than the value of -12 because -9 is to the __of -12 on the number line.


Less

Or
Greater

Answers

Answer:

Step-by-step explanation:

-9 is greater than the value of -12 because -9 is to the right of -12 on the number line.

if <ABC measures 100 and is inscribed in a circle O. find <BAO and <BCO​

Answers

Answer:

<BCO​ = <BAO​ = 20degrees

Step-by-step explanation:

If <ABC measures 100 and is inscribed in a circle O. find <BAO and <BCO​

To get <BAO and <BCO​, we need to get <AOC first.

From the figure, it can be seen that triangle ABC is an isosceles trinagle. Hence;

<BAC + <BCA + 100 = 180

Since <BAC = <BCA

<BAC + <BAC = 180 - 100

2<BAC = 80

<BAC = 80/2

<BAC = 40

Also;

<BAO = <BCO​ and <BAO = <BAC/2

<BAO = 40/2 = <BCO​

Hence <BCO​ = <BAO​ = 20degrees

Find the highest common factor (HCF) for 36, 84 and 108 ​

Answers

Answer:

Check your question one time

Determine the measure of ZA.

45.6°
57.7°
55.2°
32.3°

Answers

Step-by-step explanation:

Cos A = 40^2 + 25^2- 34^2 ÷ (2×40×25)

= 200+625-1156 ÷ (2000)

= 1069 ÷2000

Cos A = 0.5345

A= cos inverse 0.5345

A = 57.7

Answer:

57.7

Step-by-step explanation:

took the test

Hey is there any chance anyone could help me with this question ASAP?? Tysm :)

Answers

Answer:

6

Step-by-step explanation:

A = π. r²

36π = π. r²

eliminate π, we get:

36=r²

r=√36

r = 6

Answer:

r = 6

Step-by-step explanation:

The area (A) of a circle is calculated as

A = πr² ( r is the radius )

Given A = 36π , then

πr² = 36π ( divide both sides by π )

r² = 36 ( take the square root of both sides )

r = [tex]\sqrt{36}[/tex] = 6

Which of the following shows 5x + 17 + 8x – 9 + 2y in simplest terms?

Answers

Answer:

5x+8x+17-9+2y

13x+8+2y

Answer:

13x+8+2y

Step-by-step explanation:

5x+8x=13x

17–9=8

2y=2y

The temperature of a cup of coffee varies according to Newton's Law of Cooling: -"dT/dt=k(T-A), where is the temperature of the coffee, A is the room temperature, and k is a positive
constant. If the coffee cools from 100°C to 90°C in 1 minute at a room temperature of 25*C, find the temperature, to the nearest degree Celsius of the coffee after 4 minutes,
74
67
60
42

Answers

Answer:

B) 67°C.

Step-by-step explanation:

Newton's Law of Cooling is given by:

[tex]\displaystyle \frac{dT}{dt}=k(T-A)[/tex]

Where T is the temperature of the coffee, A is the room temperature, and k is a positive constant.

We are given that the coffee cools from 100°C to 90°C in one minute at a room temperature A of 25°C.

And we want to find the temperature of the coffee after four minutes.

First, solve the differential equation. Multiply both sides by dt and divide both sides by (T - A). Hence:

[tex]\displaystyle \frac{dT}{T-A}=k\, dt[/tex]

Take the integral of both sides:

[tex]\displaystyle \int \frac{dT}{T-A}=\int k\, dt[/tex]

Integrate:

[tex]\displaystyle \ln\left|T-A\right| = kt+C[/tex]

Raise both sides to e:

[tex]|T-A|=e^{kt+C}=Ce^{kt}[/tex]

The temperature of the coffee T will always be greater than or equal to the room temperature A. Thus, we can remove the absolute value:

[tex]\displaystyle T=Ce^{kt}+A[/tex]

We are given that A = 25. Hence:

[tex]\displaystyle T=Ce^{kt}+25[/tex]

Since the coffee cools from 100°C to 90°C, the initial temperature of the coffee was 100°C. Thus, when t = 0,T = 100:

[tex]100=Ce^{k(0)}+25\Rightarrow C=75[/tex]

Hence:

[tex]T=75e^{kt}+25[/tex]

We are given that the coffee cools from 100°C to 90°C after one minute at a room temperature of 25°C.

So, T = 90 given that t = 1. Substitute:

[tex]90=75e^{k(1)}+25[/tex]

Solve for k:

[tex]\displaystyle e^k=\frac{13}{15}\Rightarrow k=\ln\left(\frac{13}{15}\right)[/tex]

Therefore:

[tex]\displaystyle T=75e^{\ln({}^{13}\! /\!{}_{15})t}+25[/tex]

Then after four minutes, the temperature of the coffee will be:

[tex]\displaystyle \begin{aligned} \displaystyle T&=75e^{\ln({}^{13}\! /\!{}_{15})(4)}+25\\\\&\approx 67^\circ\text{C}\end{aligned}[/tex]

Hence, our answer is B.

Nadia is mountain climbing. She started at an altitude of 19.26 feet below sea level and then changed her altitude by climbing a total of 5,437.8 feet up from her initial position. What was Nadia’s altitude at the end of her climb?

Answers

Answer:

5418.54 ft

Step-by-step explanation:

So sea level is 0, okay? So Nadia (her name is more than 3 letters and I'm lazy so from now on she'll be reffered to as "N") is at -19. 26 ft. N goes up 5,437.8 ft, so we add this value on.

-19. 26+ 5437.8= 5418.54

Now just add on the units!

Hope this helps!

Answer:

Answer:

5418.54 ft

Step-by-step explanation:

Answer:

5418.54 ft

Step-by-step explanation:

So sea level is 0, okay? So Nadia (her name is more than 3 letters and I'm lazy so from now on she'll be reffered to as "N") is at -19. 26 ft. N goes up 5,437.8 ft, so we add this value on.

-19. 26+ 5437.8= 5418.54

pplied for a job as a cashier at a hardware store. As part of her employment screening, she was asked to take an exam that had general mathematical aptitude questions in it. This type of selection test is referred to as a

Answers

Answer:

cognitive ability

Step-by-step explanation:

There are 3 feet in 1 yard. This is equivalent to 12 feet in 4 yards. Which proportion can be used to represent this?
12th
$
12
Save and Exit
Next
Submit
Mark this and return

Answers

Answer:

divide 3 by the amount of feet to get to the yards

Step-by-step explanation:

For every three feet, there's always one yard

so to do this only given feet, we'll need to divide the amount of feet by 3 to represent the amount of yards

Inventor A had 630 inventions, 600% more than the number of inventions inventor B had. How many inventions did B have?

Answers

Answer:

105 inventions

Step-by-step explanation:

Find how many inventions Inventor B had by dividing 630 by 6:

630/6

= 105

So, Inventor B had 105 inventions

Small Manufacturing Company has a standard overhead rate of $42 per hour. The labor rate is $20 per hour. Overhead is applied based on direct labor hours. Jobs B-1 and B-2 were completed during the month of March. Small incurred 140 hours of indirect labor during the month

Answers

The question is incomplete. The complete question is :

Small Manufacturing Company has a standard overhead rate of $42 per hour. The labor rate is $15 per hour. Overhead is applied based on direct labor hours. Jobs B-1 and B-2 were completed during the month of March. Small incurred 140 hours of indirect labor during the month. Job B-1 used 82 direct labor hours and $3650 worth of direct material used. Job B-2 used 130 direct labor hours and $2,900 worth of direct material. What is the total cost of job B-2? Round to closest whole dollar (no cents).

Solution :

Particulars                                                        Job B-2

Direct material used                                        $ 2,900

Add : Direct labor cost (130 hours x $15)       $ 1,950

Add : overhead cost (130 hours x $ 42)         $ 5,460  

Total Cost of Job B-2                                      $ 10,310

Therefore, the total cost of the Job B-2 is $ 10,310.

What should you substitute for y in the bottom equation to solve the system by the substitution method?

A. y=3x+15

B. y =-x-5

C. y=x+5

D. y=-3-15

Answers

Answer: y= -x-5


Step 1: Determine which equation to change

Since we will be using the substitution method, we will need to substitute y from the bottom equation with information from the top equation. This will be the equation we change.

Step 2: Change the equation

Let’s rewrite the equation we will be working with.

3x+3y= -15

Our goal will be to get y alone on the left side. To start, let’s subtract 3x from both sides, leaving only 3y on the left side.

3x+3y= -15
3y= -3x-15

Now we need to get y completely alone by eliminating the coefficient of 3. Let’s do this by dividing each term by 3.

3y= -3x-15
y= -x-5


This is your answer. Hope this helps! Comment below for more questions.

WILL GIVE 50 POINTS PLS RESPOND FAST!!!!
The table below shows the radius y, I’m centimeters, created by grouping algae in x days:
Time (x) 2 4 6 8
(Days)
Radius(y) 4 7 10 14
(cm)

Part A: what is the most likely value of the correlation coefficient of the data in the table? Based on the correlation coefficient, describe the relationship between time and radius of the algae. [choose the value of the correlation coefficient from 1, 0.97, 0.5, 0.2.]
Part B: what is the value of the slope of the graph of radius versus time between 6 and 8 days, and what does the slope represent?
Part C: does the data in the table represent correlation or causation? Explain your answer.

Answers

Answer:

wait whats your question

Step-by-step explanation:

what is 0.7dm to nm is? please help asap​

Answers

Step-by-step explanation:

0.7 decimeter =

70,000,000 nanometers

The answer is 70,00,000 hope this helps you out :)

The incubation time for Rhode Island Red chicks is normally distributed with mean of 22 days and standard deviation of approximately 3 days. Of 1000 eggs are being incubated, how many chicks do we expect will hatch in 19 to 28 days

Answers

Answer:

We should expect 818 chicks to hatch in 19 to 28 days

Step-by-step explanation:

Normal Probability Distribution

Problems of normal distributions can be solved using the z-score formula.

In a set with mean [tex]\mu[/tex] and standard deviation [tex]\sigma[/tex], the z-score of a measure X is given by:

[tex]Z = \frac{X - \mu}{\sigma}[/tex]

The Z-score measures how many standard deviations the measure is from the mean. After finding the Z-score, we look at the z-score table and find the p-value associated with this z-score. This p-value is the probability that the value of the measure is smaller than X, that is, the percentile of X. Subtracting 1 by the p-value, we get the probability that the value of the measure is greater than X.

Normally distributed with mean of 22 days and standard deviation of approximately 3 days.

This means that [tex]\mu = 22, \sigma = 3[/tex]

Proportion between 19 and 28 days:

p-value of Z when X = 28 subtracted by the p-value of Z when X = 19.

X = 28

[tex]Z = \frac{X - \mu}{\sigma}[/tex]

[tex]Z = \frac{28 - 22}{3}[/tex]

[tex]Z = 2[/tex]

[tex]Z = 2[/tex] has a p-value of 0.977.

X = 19

[tex]Z = \frac{X - \mu}{\sigma}[/tex]

[tex]Z = \frac{19 - 22}{3}[/tex]

[tex]Z = -1[/tex]

[tex]Z = -1[/tex] has a p-value of 0.159.

0.977 - 0.159 = 0.818

Out of 1000:

0.818*1000 = 818

We should expect 818 chicks to hatch in 19 to 28 days

what is the average number of students who like cookies, chips, and crackers?
Cookies=10
Chips=3
Crackers=2

Answers

Answer:

m = 15/3 = 5 is the mean

Step-by-step explanation:

Answer:

5 Students But divide by 15/3 to  5

Step-by-step explanation:

What does Average mean?

1. a number expressing the central or typical value in a set of data, in particular the mode, median, or (most commonly) the mean, which is calculated by dividing the sum of the values in the set by their number.

So, to find the answer just ADD 10 + 2 + 3 = ?

10 + 2 = 12 So put that to the side for now.

Now, 10 + 2 = 12 + 3 = 15

So the total number of students who like cookies, chips,  and crackers are 15.

Now do 15/3 to get 5.

A plane gets an average of 25 miles per gallon when it is traveling 500 miles per hour. The plane has 15,000 gallons of gas at the beginning of a trip and travels at an average speed of 500 miles per hour. Which of the following functions f models the number of gallons of gas remaining in the tank t hours after the trip begins?

(A) f = 15000 + (25t/500)

(B) f = 15000 - (25t/500)

(C) f = 15000 - (500t/25)

(D) f = 15000 - 25t

(E) f = 25t

Answers

Answer:

D; f = 15,000 - 25t

Step-by-step explanation:

From the question;

per gallon, the number of miles traveled is 25, given traveling speed is 500 miles per hour

So after t hours, if traveling at 500 miles per hour, the amount of fuel expended will be 25 * t = 25t gallons

So, to get the amount remaining, we subtract 25t from what we have at the start of the trip

Mathematically, we have this as;

f = 15,000 - 25t

Find the missing side of the right triangle if two sides are 6, and 10

Answers

approx. 11.661… or 2 • sr of 35

Hope this helps!!

(If you have any questions, feel free to ask in the comments, If you found my answer helpful, please consider marking my answer brainliest, thanks!!)

Other Questions
Urgenteeeeeeeeeeeeeeeeeeeeeeeeeeeeeeeeeeeeeeee 2m^2-5m-3=0 by factorization Suppose you have $1750 in your savings account at the end of a certain period of time. You invested $1500 at a 3.72% simple annual interest rate. How long, in years, was your money invested? As permitted by state law, a large city in the state adopted an ordinance legalizing slot machines in shopping malls within the city. Several prominent city residents were upset by the new ordinance because gambling violates one of the main tenets of their religion. Seeking relief, the citizens contacted their representative in Congress and asked the representative to sponsor a bill making it illegal to place gambling machines in shopping malls throughout the country. The representative sponsored such a bill. Congress made a factual finding that the activity regulated has a substantial economic effect on interstate commerce and passed the statute. If the statute banning gambling machines in shopping malls is challenged on constitutional grounds by a proper plaintiff in federal court, would the court likely uphold the statute Compute ????????, where ????=????2????+5????, ????=2????+????+3????. (Write your solution using the standard basis vectors ????, ????, and ????. Use symbolic notation and fractions where needed.) PLEASE HELP!!!! WILL GIVE BRAINLIEST!!!! How did Alfred Thayer Mahan believe the United States could best secure itsinterests?A. With missionary efforts to expand Christian ideologyB. With political alliances to spread democratic principlesC. With economic colonies to harvest raw materials and supplymarketsO D. With a strong navy to protect international trade If Susan can read 40 pages in 12 minutes, how many pages can she read in 50 minutes? The key on a map says 2 inches equals 7 miles. If two cities are9 inches apart on the map, how far apart are they in real life? In recent decades there have been fewer trade blocs established to promote regional economic integration. True False Determine the molarity of the sodium ions when 78.0 g Na2S is dissolved in water for a final volume of 1.0 L. If elasticity of demand is 0.6, elasticity of supply is 0.6, and a 20% excise tax is levied on the good:________a. The fraction of the tax borne by consumers will be 0.5. b. The fraction of the tax borne by suppliers will be 0.6 c. The fraction of the tax borne by consumers will be 0.6. d. The fraction of the tax borne by suppliers will be 0.3. How is mathematical thinking established? How can we have mathematical thinking? what kind of food preserve for curing Here is the setup for a non-traditional casino game: You draw a card from a well shuffled full deck and if the card is a king you win $100. The game costs $2 to play and you decide to play the game until you win the $100. Each time you draw a card you pay $2, and if the card is not a king, the card is put back in the deck, and the deck is reshuffled. How much money should you expect to spend on this game? Identify each of the following features as applying more to job order operations, process operations or both job order and process operations.1. Cost object is a process. 2. Measures unit costs only at period-end. 3. Uses indirect costs. 4. Transfers costs between Work in Process Inventory accounts. 5. Uses only one Work in Process account. 6. Uses materials, labor, and overhead costs. All body organs are vital to lifetruefalse You get GPS units from two manufacturers, A and B. You get 43% of your units from A and 57% of your units from B. In the past, 2% of the units from A have been defective, and 1.5% of the units from B have been defective. Assuming this holds true, if a GPS unit is found to be defective what is the probability that it came from manufacturer A (think Bayes Theorem AND round to two decimal places) If you buy an amateur-sized reflecting telescope, say around 10 inches (25cm) aperture, it'll have something in it that sends the gathered starlight out the side of the telescope tube. What do we call this thing Which of the following is an autotroph? Plzzzz HelpThe cost (in dollars) of buying x pounds of a party product is given by the function C(x) = 10x + 300. Suppose, for budgetary reasons; you can't spend more than $2100 on this product. You can spend less, but you have to buy at least 50 pounds. In this situation, what is the domain of this function?